PLEASE HELP OUT! The lengths of two sides of a right triangle are given. Find the length of the third side. Round to the nearest tenth if necessary.

legs: 25 in. and 11 in.

A. 25.2 in.

B. 21.6 in.

C. 27.3 in.

D. 22.4 in.








Answers

Answer 1
I believe the answer is B

Related Questions

Which function can be used to find the number of ounces of coffee in the can based on its radius?

Answers

Answer:

A

Step-by-step explanation:

The answer is A I hope it helped!

What is the distance between (6, 2) and (-3,-2)?
09
03
O V97
05

Answers

The answer is (9, 4)

Answer:

√ 97

Decimal Form:

9.84885780 …

A customer states that she was shorted in change by receiving only 23 cents change from $3.00
dollars instead of 32 cents. How would you handle this?

Answers

Answer:

First, I would check and see if she was correct. If she wasnt, I would simply show her why she was incorrect. If I was to accidently short her, I would give her the money and move on with my day. if she refused to leave after showing her that I gave her the correct amount of money, she would probably call for the manager. I'd get the manager, explain what happened, and the annoying customer would most likely get kicked out of the store.

(P.S: sorry if it was a little confusing lol)

y=6x+15 what is y, when x=5

Answers

Answer:

Answer:45

6 x 5+15=45

Step-by-step explanation:

Hope this helped!

Answer:

Look at explanation

Step-by-step explanation:

First, plug in x=5 into the equation y=6x+15 for x.

[tex]y=6(5)+15[/tex]

Solve.

[tex]y=6(5)+15\\=30+15\\=45\\y=45[/tex]

There are 345 fifth graders enrolled at Wilson Middle School and 15 fifth-grade classrooms. How many students are in each class if each class has the soame number of students?

Answers

Answer:23

Step-by-step explanation:

Literally just division

Answer:

23

Step-by-step explanation:

345\15=23

In the diagram, which two angles are alternate interior angles with angle 14?

4 lines intersect to form 16 angles. The angles created, clockwise from top left are 1, 2, 3, 4; 5, 6, 7, 8; 13, 14, 15, 16; 9, 10, 11, 12.
Angle 4 and Angle 12
Angle 3 and Angle 9
Angle 2 and Angle 10
Angle 12 and Angle 15

Answers

Answer:

Angle 4 and Angle 12

Step-by-step explanation:

Angles 4 and angles 12 are alternate interior angles with 14.

Angle 4 is alternate interior with angle 14, and angle 12 is alternate interior with angle 14.

The person that answered above me is wrong. But there is some truth to his answer, because angle 3 is alternate interior with angle 9, but they are NOT alternate interior WITH angle 14.

That is why many people answering this question fail. The question says:

In the diagram, which two angle ARE alternate interior angles WITH angle 14?

To finally answer the question completely, angles 4 and 12 ARE alternate interior angles WITH angle 14.

Therefore the guy above me is 100% wrong.

Hence, the right option is (A).

1.
Which equation in point-slope form contains the point (-2, -6) and has slope -4?
y - 2 = -4(X-6)
y + 2 = -4(x + 6)
.
y + 6 = -4(x + 2)
y-6 = -4(x - 2)

Answers

Answer:

Step-by-step explanation:

y + 6 = -4(x + 2) is the point-slope form

y + 6 = -4x - 8

y = -4x - 14

Suzy is shopping at a book sale. The table shows prices for paperbacks.

A two-column table with 3 rows. Column 1 is labeled Books (x) with entries 4, 6, 8. Column 2 is labeled Cost (y) with entries 6, 9, 12.

These points are graphed. Notice the pattern of the points. What is the rate?

The cost is $
for every 2 books.

Answers

Answer:

3 bucks

Step-by-step explanation:

The rate of change is 2 and the cost of two books is $3.

What are lines and their slopes?

We know lines have various types of equations, the general type is

Ax + By + C = 0, and the equation of a line in slope-intercept form is

y = mx + b.

Where slope = m and b = y-intercept.

the slope is the rate of change of the y-axis with respect to the x-axis and the y-intercept is the (0,b) where the line intersects the y-axis at x = 0.

Given, Suzy is shopping at a book sale. The table shows prices for paperbacks.

The points are, (4, 6), (6, 8), and (8, 12).

Slope(m) = (12 - 8)/(8 - 6).

Slope(m) = 4/2.

Slope(m) = 2.

Now, 4 books cost $6, hence 2 books cost $3.

learn more about lines and slopes here :

https://brainly.com/question/3605446

#SPJ3

1 gal 3 c= _fl oz

Convert the unit of capacity

Answers

Answer:

152 fluid ounces

Step-by-step explanation:

there are 128 fluid ounces in 1 gallon. there are 24 fluid ounces in 3 cups. 128+24 is 152. :)

help asap will mark brainliest

Answers

Answer:

Step-by-step explanation:

x = 105

the angle vertically opposite to x is 105 as it is corresponding to 105

and corresponding angles are equal

since vertically opposite angles are equal too, then x = 105 too

Answer:

Step-by-step explanation:

x = 105 {alternate exterior angles are equal}

When two parallel lines are intersected by the transversal, alternate exterior angles are equal

PLEASE HELP NOW!! BRAINLIEST

Point P(-3,-4) is rotated 270° clockwise about the origin.
What are the coordinates of its image after this transformation?

Answers

Answer:

(4,-3)

Step-by-step explanation:

If you graph this point, then it will be in the 3rd quadrant.

A 180 degree rotation will put it in the 1st quadrant.

And the extra 90 degrees will put in in the 4th quadrant, so we know that in (x,y), x is positive, and y is negative. then if you theoretically rotate it, since you are rotating it 90 degrees, 3 times, you will have to switch the x and y, since 3 is an odd number. so the final answer is (4,-3).

The coordinates of the given image after this transformation will be (4,-3).

What is transformation?

A transformation is a general term for four specific ways to manipulate the shape and/or position of a point, a line, or geometric figure.

Given that, Point P(-3,-4) is rotated 270° clockwise about the origin.

When graph this point, then it will be in the 3rd quadrant.

A 180° rotation will put it in the 1st quadrant.

And the extra 90° will put in the 4th quadrant, so we know that in (x, y), x is positive, and y is negative. then if you theoretically rotate it, since you are rotating it 90°, 3 times, you will have to switch the x and y, since 3 is an odd number.

Hence, The coordinates of the given image after this transformation will be (4,-3).

For more references on transformation, click;

https://brainly.com/question/11709244

#SPJ2

which of the following are properties of the circumference of a triangle

Answers

Answer:

angle, length height

Step-by-step explanation:

The circumference of a triangle are the lengths of all three sides added together. If you ask yourself what are the properties you need to measre the lengths of a side.

Ming took a cab across town. His fare was $22, and he leaves an 18%tip.
What is the total amount Ming pays the cab driver?

Answers

He payed a total of 25.96 because the tip was 3.96

Answer:

$25.96

Step-by-step explanation:

take 18% of 22, which is 3.96. Add it to 22 and that's the amount Ming payed!

Todd has created the following diagram to prove the Pythagorean theorem. He drew (triangle)ABC with leg lengths a and b and hypotenuse length c. Then he drew altitude CZ to the hypotenuse. He determines that c/a=a/z and c/b=b/y. Then he determines that cx=a^2 and cy=b^2. What is the next step in proving the theorem?

Answers

Answer:

cx + cy = a² + b²

Step-by-step explanation:

Next step is adding up cx and cy:

cx + cy = a² + b²

Then followed by:

c(x+y) =  a² + b²c² =  a² + b²

Answer: cx + cy = a² + b²

Step-by-step explanation:

Solve 1/2n +3 < 5.
Which graph shows the solutions?

I WILL GIVE BRAINLY

Answers

Answer:

Third one

Step-by-step explanation:

½n + 3 < 5

½n < 2

n < 4

Open circle at 4 because 4 in not included, arrow pointing towards left because it is less than 4

The option c is solution of given equation

What is graph?

the collection of all points whose coordinates satisfy a given relation (such as a function)

According to the question,

½n + 3 < 5

½n < 2

n < 4

Open circle at 4 because 4 in not included, arrow pointing towards left because it is less than 4

Hence, option c is correct

To learn more related to this topic from here

https://brainly.in/question/6893068

#SPJ2


Adding and subtracting radicals

Answers

the correct answer would be 0 i believe

Find the are. Will mark brainliest.

Answers

Answer:

4.44

Step-by-step explanation:

To find the area of a triangle, multiply the base by the height, and then divide by 2.2.4 x 3.7 = 8.88next i divide by 28.88 / 2 = 4.44so my answer is 4.44

Hope this helpsask questions if it is wrongBrainliest plz

I WILL GIVE BRAINLIST!! Two joggers run 9 miles north then 3 miles west.What is the shortest distance, to the nearest tenth of a mile, they must travel to return to their starting point?

Answers

i’m not sure but i think it’s 9.5

Answer:

9.5 is your answer

Step-by-step explanation:

9^2 + 3^2 =

81 + 9 =

[tex]\sqrt{90}[/tex] = 9.5

Michael earns 12% commission on cell phone sales. He sold $549 worth of cell phones this week. How much did he earn in commission?

Answers

Answer:

Michael earns $69.88 in commission.

Step-by-step explanation:

549*0.12=69.88

Estimate the number of boys at Hillview.

Answers

Answer:

800 boys

Step-by-step explanation:

The figure below shows the quotient of Fraction 3 over 4 divided by Fraction 3 over 8 .

Rectangle divided into eight equal parts, where the first three-part is shaded dark representing three-eighths, the next three parts are shaded light to complete the three-fourths, and the last two parts are not shaded

The quotient is ____.

Answers

3/4 divided by 3/8 is 2
The correct answer is 2

i need help!!!
y=6x
y=42

simplify the answer as much as possible

Answers

Answer: X=7

Explanation: because 6x7=42 and 42 is y
Answer & Explanation:

1. Set both of the equations equal to each other to find x.

2. 6x=42

3. Divide both sides by 6.

4. X=7

5. Now let’s find y

6. y=42

7. Check your work:
Plug in the value of x into this equation. y=6x

We know that x=7, so y=6(7)= 42

This means that x=7 and y=42

I hope this helps you!!!!

Below is the graph of a trigonometric function. It has a minimum point at (- 10.4, - 9.8) and a maximum point at (- 3.6, - 1.2) . What is the period of the function?

Answers

Answer:

13.6

Step-by-step explanation:

The period is double of the distance (on the x-axis) between the maximum and the minimum. In this case, the x-coordinate of the minimum is -10.4, the x-coordinate of the maximum is -3.6, the distance is -3.6 - (-10.4) = 6.8, and the period is 2*6.8 = 13.6

Answer:

Step-by-step explanation:

4.3 unit from by khan academy

A cylinder and its dimensions are shown. Which equation can be used to find V, the volume of the cylinder in cubic centimeters?
A) V = π(3.5h)2
B) V = π(7h)2
C) V = π(3.5)2h
D) V = π(7)2h

Answers

Answer:

The volume of a cylinder is given by the equation:

V = π(R)^^2*h

where π = 3.14, R is the radius and h is the height.

If in the image we see that the radius is 7, then the correct option is D.

If in the image we shee that the DIAMETER is 7 (a linea that goes from end to end of the circle and goes trough the middle) then we know that the radus is half of the diameter, 7/2 = 3.5

in this case he correct option is C.

Answer:

There is no image included, so I cannot give a direct answer. However, you should be able to figure out the answer after my explanation.

Step-by-step explanation:

‼️now pay attention in order to get this question correct‼️

Well, this is a simple problem, since you don’t necessarily need to solve it. First, you need to know the formula of volume for a cylinder. This is: V = πr²h.

Figure out π. Π is a number too long to be written, so we found it up to 3.14.

π = 3.14

Now figure out the radius, for r. This, isn’t my problem to solve because I don’t have a picture of the cylinder so I cant determine so.

The equations don’t specify any height written, so we don’t have to solve for h yet.

This is your job now. So far, I’ve only figured out π, now find the radius, take that number, and fit it in the parentheses. V = π(_)²h. That’s it. Now look at the options, and pick the option that matches what you’ve already written.

I hope I’ve helped, taking time to write this... but I hope you wouldn’t mind brainliest lol c: take care and have a nice day!

Which figure has an orthocenter outside the triangle? A triangle has angle measures of 30, 75, and 75 degrees. A triangle has angle measures of 54, 56, and 74 degrees. A triangle has angle measures of 45, 110, and 25 degrees. A triangle has angle measures of 90, 48, and 42 degrees. plzzzz help

Answers

Answer:

A triangle has angle measures of 45, 110, and 25 degrees would has an orthocenter outside the triangle.

Hope this helps!

:)

Answer:

The Third option is correct.

Step-by-step explanation: just got it right on edge

Look at the figure below. Which of the following statements could be a step in a proof that vertical
angles are congruent?
APC = DPB because angles formed by the
same pair of intersecting lines are congruent to each
other.
ZAPC ZDPB because angles complementary
to the same angle are congruent to each other.
ZAPC ZDPB because angles adjacent to the
same angle are congruent to each other.
ZAPC ZDPB because angles supplementary to
the same angle are congruent to each other.

Answers

Answer:

ZAPC ZDPB because angles adjacent to the

same angle are congruent to each other.

Step-by-step explanation:

Jared makes $133 per hour, but he loses 20% of his pay for a day if his boss catches him on the internet. If Jared works for 8 hours a day for 5 days, and his boss only catches him on the internet once how much money will he make in the five days?

Answers

Answer:

$5107.20

Step-by-step explanation:

His pay after he was caught once

133x 8=1064

1064x 20% = 212.8

1064-212.8=851.2

1064x 4= 4256

4256+ 851.2=5107.2

The taco truck sells tacos for and burritos for . The number of items sold on a Tuesday is with a total income of . How many tacos were sold?

Answers

Answer:

You didnt say the price otherwise how am i suppose to give you the answer

Step-by-step explanation:

Which of the following shows the correct solution steps and solution to

3x-2= -11

Answers

Answer:

the third one is correct

Step-by-step explanation:

The third one is correct

help me asap right answer gets most brainlyest

Answers

Answer: The answer is 1/4th :)

You add all the variables together then add B & C ... well your answer is 1/4
Other Questions
The function fff is given in three equivalent forms. Which form most quickly reveals the zeros (or "roots") of the function? Choose 1 answer: Choose 1 answer: (Choice A) A f(x)=-3(x-2)^2+27f(x)=3(x2) 2 +27f, (, x, ), equals, minus, 3, (, x, minus, 2, ), squared, plus, 27 (Choice B) B f(x)=-3(x+1)(x-5)f(x)=3(x+1)(x5)f, (, x, ), equals, minus, 3, (, x, plus, 1, ), (, x, minus, 5, )(Choice C) C f(x)=-3x^2+12x+15f(x)=3x 2 +12x+15f, (, x, ), equals, minus, 3, x, squared, plus, 12, x, plus, 15 Write one of the zeros. xxx = Write a single paragraph informational essay on the following question: As a seventh grader, what are the challenges of fitting in with your peers? Why do some music artists believe that there music is being devalued What is the purpose of boo radley in to kill a mocking bird Of all 25 marbles in a bag, 3 of the marbles are white. What is the probability that a white marble will be randomly selected from the bag without looking? Write your answer as a percent. Two number cubes each have sides that are labeled 1 to 6. Jude rolls the 2 number cubes. What is the probability that the sum of the numbers oncubes will equal 4? factorise 15x- 10x^3 The table shows the heights of 40 students in a class.-Height (h)in cm-120 < t < 124124 < t < 128128 < t < 132132 Question 17 In the Haber reaction, patented by German chemist Fritz Haber in 1908, dinitrogen gas combines with dihydrogen gas to produce gaseous ammonia. This reaction is now the first step taken to make most of the world's fertilizer. Suppose a chemical engineer studying a new catalyst for the Haber reaction finds that 786. liters per second of dinitrogen are consumed when the reaction is run at 222.C and 0.35atm. Calculate the rate at which ammonia is being produced. Give your answer in kilograms per second. Round your answer to 2 significant digits. The small city-country located at number 6 on the map above is __________.A.SingaporeB.MalaysiaC.BangkokD.Jakarta A firearms company is testing a new model of rifle by firing a 7.50-g lead bullet into a block of wood having a mass of 17.5 kg. The bullet embeds into the block and the collision generates heat. As a consequence, the temperature rises by 0.040C, as measured with a high-precision thermometer. Assuming that all the kinetic energy of the bullet goes into heating the system, what is the bullets speed when it enters the block? The initial temperatures of bullet and wooden block can be considered identical and the specific heats of lead and wood are cPb = 130 J/(kg C) and c wood = 1700 J/(kg C), respectively. Which of the following are examples of short-term, human-induced environmental changes? Select three answer choices. mass extinctionglobal warmingdeforestationozone layer destructionpollution Select the noun or pronoun that is the object of the preposition,Never be afraid of failure. Which equation is NOT an example of a linear function?A)y = 9 - 2xB)y = 6/XC) y = x/2 + 9D) y = 5/6x - 8 A square pyramid is sliced parallel to the base and halfway up the pyramid. Which of these describes the cross section?A square smaller than the baseA quadrilateral that is not a squareA square the same size as the baseA triangle with a height the same as the pyramid triangle xyz is similar to triangle abc solve for k What is the area of triangle ABC?O 2.5 square unitsO 3 square unitsO 3.5 square unitsO4 square units who was one of the main authors of the Declaration ofIndependence? If a figure is translated 5 units down and 3 units right, how do you represent it algebraically Explain the three steps when using distributive property to write an equivalent expression.